Menu Close

prove-it-times-n-4-4-4-4-lt-3-




Question Number 192062 by mehdee42 last updated on 07/May/23
prove it :      times_n   ;   (√(4+(√(4+(√(4+...+(√4)))))  )) < 3
$${prove}\:{it}\::\: \\ $$$$\:\:\:{times\_n}\:\:\:;\:\:\:\sqrt{\mathrm{4}+\sqrt{\mathrm{4}+\sqrt{\mathrm{4}+…+\sqrt{\mathrm{4}}}}\:\:}\:<\:\mathrm{3} \\ $$
Commented by ajfour last updated on 08/May/23
but 4> 3      how can this be...  really no use arguing!
$${but}\:\mathrm{4}>\:\mathrm{3}\:\:\:\: \\ $$$${how}\:{can}\:{this}\:{be}… \\ $$$${really}\:{no}\:{use}\:{arguing}! \\ $$
Commented by ajfour last updated on 08/May/23
sometimes it just really dont make no sense et al.
Commented by mehdee42 last updated on 08/May/23
pay attention to question     that is ((√4)<3 ) right , not what you wrote (4>3)
$${pay}\:{attention}\:{to}\:{question}\:\:\: \\ $$$${that}\:{is}\:\left(\sqrt{\mathrm{4}}<\mathrm{3}\:\right)\:{right}\:,\:{not}\:{what}\:{you}\:{wrote}\:\left(\mathrm{4}>\mathrm{3}\right) \\ $$
Answered by Frix last updated on 07/May/23
x=(√(4+(√(4+(√(4+...))))))>0  x=(√(4+x))  x^2 −x−4=0  x=((1+(√(17)))/2)  ((1+(√(17)))/2)<3  1+(√(17))<6  (√(17))<5  17<25 true
$${x}=\sqrt{\mathrm{4}+\sqrt{\mathrm{4}+\sqrt{\mathrm{4}+…}}}>\mathrm{0} \\ $$$${x}=\sqrt{\mathrm{4}+{x}} \\ $$$${x}^{\mathrm{2}} −{x}−\mathrm{4}=\mathrm{0} \\ $$$${x}=\frac{\mathrm{1}+\sqrt{\mathrm{17}}}{\mathrm{2}} \\ $$$$\frac{\mathrm{1}+\sqrt{\mathrm{17}}}{\mathrm{2}}<\mathrm{3} \\ $$$$\mathrm{1}+\sqrt{\mathrm{17}}<\mathrm{6} \\ $$$$\sqrt{\mathrm{17}}<\mathrm{5} \\ $$$$\mathrm{17}<\mathrm{25}\:\mathrm{true} \\ $$
Commented by mehdee42 last updated on 07/May/23
pay  attention :  (√(4+(√(4+(√(4+....)))))) ≠ (√(4+(√(4+(√(4+...(√4)))))))   ⇒ if   x=(√(4+(√(4+(√(4+...(√4)))))))⇏x^2 =4+x  the expression on the left contains the infinitive of the   sentence .while the number of  sentenes   in thr right experession is finit.
$${pay}\:\:{attention}\:: \\ $$$$\sqrt{\mathrm{4}+\sqrt{\mathrm{4}+\sqrt{\mathrm{4}+….}}}\:\neq\:\sqrt{\mathrm{4}+\sqrt{\mathrm{4}+\sqrt{\mathrm{4}+…\sqrt{\mathrm{4}}}}}\: \\ $$$$\Rightarrow\:{if}\:\:\:{x}=\sqrt{\mathrm{4}+\sqrt{\mathrm{4}+\sqrt{\mathrm{4}+…\sqrt{\mathrm{4}}}}}\nRightarrow{x}^{\mathrm{2}} =\mathrm{4}+{x} \\ $$$${the}\:{expression}\:{on}\:{the}\:{left}\:{contains}\:{the}\:{infinitive}\:{of}\:{the}\: \\ $$$${sentence}\:.{while}\:{the}\:{number}\:{of}\:\:{sentenes}\: \\ $$$${in}\:{thr}\:{right}\:{experession}\:{is}\:{finit}. \\ $$
Commented by Frix last updated on 07/May/23
x=((1+(√(17)))/2)  (√4)<(√(4+(√4)))<(√(4+(√(4+(√4)))))<...<x<3
$${x}=\frac{\mathrm{1}+\sqrt{\mathrm{17}}}{\mathrm{2}} \\ $$$$\sqrt{\mathrm{4}}<\sqrt{\mathrm{4}+\sqrt{\mathrm{4}}}<\sqrt{\mathrm{4}+\sqrt{\mathrm{4}+\sqrt{\mathrm{4}}}}<…<{x}<\mathrm{3} \\ $$
Commented by mehdee42 last updated on 07/May/23
why  “...< x <3 ” ?!
$${why}\:\:“…<\:{x}\:<\mathrm{3}\:''\:?! \\ $$
Commented by AST last updated on 07/May/23
Let 4_n =(√(4+(√(4+(√(4+...+(√4))))))) (where 4 appears n  times)  4_n <4_p  when n<p...(i)  This is true since 4_n =(√(4+z)) and z increases as  n increases.  since 4_∞ <3,(i)⇒4_n <4_∞ <3  Hence,we have shown that for all n,4_n <3.
$${Let}\:\mathrm{4}_{{n}} =\sqrt{\mathrm{4}+\sqrt{\mathrm{4}+\sqrt{\mathrm{4}+…+\sqrt{\mathrm{4}}}}}\:\left({where}\:\mathrm{4}\:{appears}\:{n}\right. \\ $$$$\left.{times}\right) \\ $$$$\mathrm{4}_{{n}} <\mathrm{4}_{{p}} \:{when}\:{n}<{p}…\left({i}\right) \\ $$$${This}\:{is}\:{true}\:{since}\:\mathrm{4}_{{n}} =\sqrt{\mathrm{4}+{z}}\:{and}\:{z}\:{increases}\:{as} \\ $$$${n}\:{increases}. \\ $$$${since}\:\mathrm{4}_{\infty} <\mathrm{3},\left({i}\right)\Rightarrow\mathrm{4}_{{n}} <\mathrm{4}_{\infty} <\mathrm{3} \\ $$$${Hence},{we}\:{have}\:{shown}\:{that}\:{for}\:{all}\:{n},\mathrm{4}_{{n}} <\mathrm{3}. \\ $$
Commented by mehdee42 last updated on 07/May/23
sir  why  4_n <4_∞ <3  ??
$${sir} \\ $$$${why}\:\:\mathrm{4}_{{n}} <\mathrm{4}_{\infty} <\mathrm{3}\:\:?? \\ $$
Commented by mehdee42 last updated on 07/May/23
it can be proven by a very simple   mahematical induction metod  good luck
$${it}\:{can}\:{be}\:{proven}\:{by}\:{a}\:{very}\:{simple}\: \\ $$$${mahematical}\:{induction}\:{metod} \\ $$$${good}\:{luck} \\ $$
Answered by mehdee42 last updated on 08/May/23
answer to question number   let : p_n = n_ termes   (√(4+(√(4+(√(4+...+(√4))))) ))<3   p_1  =(√4)=2<3 ✓      i.s  p_k  = (k _ termes ) (√(4+(√(4+(√(4+....+(√4)))))))<3    i.h   p_(k+1)  =(k+1 _ termes)   (√(4+(√(4+(√(4+...+(√4))))))) <3   ?  i.r  p_(k+1) ^2  =4+(k_termes) (√(4+(√(4+(√(4+...+(√4))))) ))<4+3=7   ⇒ p_(k+1) <(√7)<3 ✓
$${answer}\:{to}\:{question}\:{number}\: \\ $$$${let}\::\:{p}_{{n}} =\:{n\_}\:{termes}\:\:\:\sqrt{\mathrm{4}+\sqrt{\mathrm{4}+\sqrt{\mathrm{4}+…+\sqrt{\mathrm{4}}}}\:}<\mathrm{3} \\ $$$$\:{p}_{\mathrm{1}} \:=\sqrt{\mathrm{4}}=\mathrm{2}<\mathrm{3}\:\checkmark\:\:\:\:\:\:{i}.{s} \\ $$$${p}_{{k}} \:=\:\left({k}\:\_\:{termes}\:\right)\:\sqrt{\mathrm{4}+\sqrt{\mathrm{4}+\sqrt{\mathrm{4}+….+\sqrt{\mathrm{4}}}}}<\mathrm{3}\:\:\:\:{i}.{h}\: \\ $$$${p}_{{k}+\mathrm{1}} \:=\left({k}+\mathrm{1}\:\_\:{termes}\right)\:\:\:\sqrt{\mathrm{4}+\sqrt{\mathrm{4}+\sqrt{\mathrm{4}+…+\sqrt{\mathrm{4}}}}}\:<\mathrm{3}\:\:\:?\:\:{i}.{r} \\ $$$${p}_{{k}+\mathrm{1}} ^{\mathrm{2}} \:=\mathrm{4}+\left({k\_termes}\right)\:\sqrt{\mathrm{4}+\sqrt{\mathrm{4}+\sqrt{\mathrm{4}+…+\sqrt{\mathrm{4}}}}\:}<\mathrm{4}+\mathrm{3}=\mathrm{7}\: \\ $$$$\Rightarrow\:{p}_{{k}+\mathrm{1}} <\sqrt{\mathrm{7}}<\mathrm{3}\:\checkmark \\ $$$$ \\ $$

Leave a Reply

Your email address will not be published. Required fields are marked *